LSAT and Law School Admissions Forum

Get expert LSAT preparation and law school admissions advice from PowerScore Test Preparation.

 Sherry001
  • Posts: 81
  • Joined: Aug 18, 2014
|
#21190
Hello;
I had trouble identifying the conclusion for this question. I did the therefore test and to me the first sentence proved to be the conclusion. This is how I broke down the stimulus.

1- the present one cannot be expanded.
2- land is more expensive in town than in the outskirts .
3- cost of acquiring a site is a significant part of the total construction cost.

Intermediate conclusion : the post office could be built more cheaply on the outskirts of town.
Conclusion : Brownleas post office must be replaced with a larger one.

Thanks for your help.
Sherry
 Lucas Moreau
PowerScore Staff
  • PowerScore Staff
  • Posts: 216
  • Joined: Dec 13, 2012
|
#21216
Hello, Sherry,

I'm glad you're practicing at breaking down the stimulus into its logical chunks. That's a good habit to get into. :)

The conclusion of this stimulus isn't that Brownlea's post office must be replaced by a larger one, though, that's one of the premises.

SINCE Brownlea's post office must be replaced by a larger one,
AND SINCE the present one cannot be expanded,
AND SINCE land is more expensive in town than in the outskirts,
AND SINCE the cost of acquiring a site is a significant part of the total construction cost,
THEREFORE the post office could be built more cheaply on the outskirts of town.

Answer choice B throws a wrench into this chain of logic by adding in the idea of a parking lot. If the post office must have a parking lot, that's more land to be bought - and if you've ever been near a commercial business, you'll know that parking lots cover much more land than a building does. ;)

Hope that helps,
Lucas Moreau
 andriana.caban
  • Posts: 142
  • Joined: Jun 23, 2017
|
#37872
Can you explain why answer choice A is incorrect?
 Francis O'Rourke
PowerScore Staff
  • PowerScore Staff
  • Posts: 471
  • Joined: Mar 10, 2017
|
#38122
Choice (A) gives us information about total cost, regardless of where in the city the new post office is built. Since the argument concludes that one location in the town is cheaper than another location in the town, and choice (A) adds a cost to both options, this answer does not weaken the conclusion. One location could still be cheaper than another, even if both options are expensive.
 MikeJones
  • Posts: 31
  • Joined: Oct 02, 2017
|
#40242
Francis O'Rourke wrote:Choice (A) gives us information about total cost, regardless of where in the city the new post office is built. Since the argument concludes that one location in the town is cheaper than another location in the town, and choice (A) adds a cost to both options, this answer does not weaken the conclusion. One location could still be cheaper than another, even if both options are expensive.
Great explanation. I'd also like to add that a "demanding citywide building code" doesn't necessarily translate to increased cost. We have no idea how much it costs to meet the code, or if it will cost anything at all. That's an extra assumption you'd have to make.
 taylorballou
  • Posts: 18
  • Joined: Feb 18, 2017
|
#40247
Hello,

Could you explain why Answer Choice C is incorrect? I felt like Answer Choice B made the unwarranted assumption that the parking lot would cost a substantial amount, whereas I guess I assumed expanding the bus routes would be substantially expensive.

Thanks,

Taylor
 Francis O'Rourke
PowerScore Staff
  • PowerScore Staff
  • Posts: 471
  • Joined: Mar 10, 2017
|
#40286
Hi Taylor,

The speaker concludes that building the post office on the outskirts of town would be cheaper than building it near the center. A bus line expansion may increase public transit costs for the city. However, the argument only concerns relative building costs for these two sites. The speaker makes no claims about the total yearly operating costs for the city, so introducing this information would not weaken the argument.

Answer choice (C) does present us with negative information about building the post office on the outskirts, but the specific negative factor it brings up is tangential to the argument. Be careful when you spot a Weaken question answer choice that gives clearly negative information. You must always ask yourself if this information is relevant to the argument or if it is out of scope.

Get the most out of your LSAT Prep Plus subscription.

Analyze and track your performance with our Testing and Analytics Package.